Why S, T and U Must Be Positive in Mandelstam Variables

In summary: I also tried working in the rest frame of one of the particles with a similar result.have u used what E_3 will be in the CM frame?have u tried going to e.g. 3's rest frame?also you can try with all m -> 0 and then add a "small" mass?t and u can be positive if the mass is not zero.
  • #1
LAHLH
409
1
Hi,

Srednicki says in Ch20, we must remember that the mandelstam variable s is positive. However s is defined as [tex]s=-(k_1+k_2)^2=(E_1+E_2)^2-(\vec{k_1+k_2}).(\vec{k_1+k_2}) [/tex]. Using metric (-+++). I can't quite see why this must be always positive? or why for that matter t and u are negative?

Also he says we get [tex] V_3(s) [/tex], from the general [tex] V_3(k_1,k_2, k_3) [/tex] by setting two of the three k's to -m^2, and the remaining one to -s. Does anyone know why these values? I would have imagined one would be [tex] sqrt(-s)=k_1+k_2 [/tex] and the other two were just left as [tex] k_1[/tex] and [tex] k_2 [/tex] for scattering in s channel kind of way.

Thanks again
 
Last edited:
Physics news on Phys.org
  • #2
just work it out,

[tex]
s=-(k_1+k_2)^2=(E_1+E_2)^2-(\vec{k_1+k_2}).(\vec{k_1+k_2})
[/tex]

write k_1 dot k_2 etc as functions of E and mass m.

[tex]s = m_1^2 + m_2^2 + 2( E_1E_2 - cos \theta \sqrt{E_1^2-m_1^2} \sqrt{E_2^2-m_2^2} \:\: ) [/tex]

then you'll see that s >= 0
 
  • #3
Thanks for the reply ansgar. Is the expression you posted postivie just becase [tex] \sqrt{E_1^2-m_1^2} < \sqrt{E_1^2}=E_1 [/tex] and sim for second factor. So even if [tex] cos(\theta)=1 [/tex], the [tex] cos \theta \sqrt{E_1^2-m_1^2} \sqrt{E_2^2-m_2^2} [/tex] term is less than [tex] E_1E_2[/tex]. Therefore the whole of s must be positive?

The way I convinced myself s was postivie before seeing you post, was looking at it in the CM frame, and using the fact s is a Lorentz scalar. In the CM frame [tex] s=(E_1+E_2)^2 [/tex] which is obviously positive.

I still have been unable to convince myself t and u are negative however, either by switching to a conveinient frame or otherwise.

Thanks again for the help
 
  • #4
LAHLH said:
Thanks for the reply ansgar. Is the expression you posted postivie just becase [tex] \sqrt{E_1^2-m_1^2} < \sqrt{E_1^2}=E_1 [/tex] and sim for second factor. So even if [tex] cos(\theta)=1 [/tex], the [tex] cos \theta \sqrt{E_1^2-m_1^2} \sqrt{E_2^2-m_2^2} [/tex] term is less than [tex] E_1E_2[/tex]. Therefore the whole of s must be positive?

The way I convinced myself s was postivie before seeing you post, was looking at it in the CM frame, and using the fact s is a Lorentz scalar. In the CM frame [tex] s=(E_1+E_2)^2 [/tex] which is obviously positive.

I still have been unable to convince myself t and u are negative however, either by switching to a conveinient frame or otherwise.

Thanks again for the help

You should not struggle with this if u want to learn QFT ;)

Yes that is also a good way to see it :)
 
  • #5
ansgar said:
You should not struggle with this if u want to learn QFT ;)

Yes that is also a good way to see it :)

Yes, yes, I know it's trivial, just tripping myself up trying to see it exactly, but think I got there from multiple directions in the end. Having said that is there an analagous way to show t and u are negative? I've tried various frames I thought might be useful without anything that has convinced me.
 
  • #6
LAHLH said:
Yes, yes, I know it's trivial, just tripping myself up trying to see it exactly, but think I got there from multiple directions in the end. Having said that is there an analagous way to show t and u are negative? I've tried various frames I thought might be useful without anything that has convinced me.

show us what u have tried
 
  • #7
Well for example [tex] t=-(k_1-k_3)^2[/tex]

I naivley tried CM frame again at first:

[tex] t=-[(E_1-E_3, 2\vec{k_1})]^2=(E_1-E_3)^2-4\vec{k^{2}_1} [/tex]

Then subbing in the usual Einstein energy momentum relation:

[tex] t=-[(E_1-E_3, 2\vec{k_1})]^2=(E_1-E_3)^2-4(E^{2}_1-m^{2}_1) [/tex]

[tex] t=E^{2}_3+4m^{2}_1-2E_3E_1-3E^{2}_1 [/tex]

I can't think of a clever way to argue why this must always be negative right now

-----------------------------

I also tried working in the rest frame of one of the particles with a similar result.
 
  • #8
have u used what E_3 will be in the CM frame?

have u tried going to e.g. 3's rest frame?

also you can try with all m -> 0 and then add a "small" mass?
 
Last edited:
  • #9
t and u can be positive if the mass is not zero.
 

Related to Why S, T and U Must Be Positive in Mandelstam Variables

1. Why must S, T, and U be positive in Mandelstam variables?

The positivity of S, T, and U in Mandelstam variables is a fundamental requirement for the consistency of quantum field theory. This condition ensures that the particles involved in a scattering process have positive energies, which is necessary for the stability of the system.

2. What happens if S, T, or U is negative in Mandelstam variables?

If any of the variables S, T, or U is negative, it would imply that at least one of the particles involved in the scattering process has negative energy. This would violate the energy-momentum conservation laws and render the calculation of amplitudes and cross-sections meaningless.

3. Can S, T, or U be equal to zero in Mandelstam variables?

No, S, T, and U cannot be equal to zero in Mandelstam variables. This is because zero values would correspond to massless particles, and the Mandelstam variables are defined in terms of the masses of the particles involved. Additionally, a zero value would also lead to a singularity in the equations, making them mathematically inconsistent.

4. How are S, T, and U related to the center-of-mass energy in Mandelstam variables?

The center-of-mass energy squared (s) is equal to the sum of S, T, and U in Mandelstam variables. This relationship is crucial for determining the maximum energy available for a scattering process and for understanding the behavior of particle interactions at high energies.

5. Are there any exceptions to the positivity requirement of S, T, and U in Mandelstam variables?

In some special cases, such as in theories with extra dimensions, the positivity requirement for S, T, and U may be relaxed. However, this is not a common occurrence and does not change the fundamental principle that these variables must be positive in most quantum field theories.

Similar threads

Replies
8
Views
1K
Replies
1
Views
2K
  • Quantum Physics
Replies
5
Views
2K
Replies
2
Views
896
  • Quantum Physics
Replies
10
Views
3K
  • Calculus and Beyond Homework Help
Replies
6
Views
1K
  • Quantum Interpretations and Foundations
Replies
9
Views
2K
Replies
8
Views
934
  • Introductory Physics Homework Help
Replies
1
Views
2K
  • Advanced Physics Homework Help
Replies
1
Views
921
Back
Top